Block 3 Pace 6 Nov 2011

Approved & Edited by ProProfs Editorial Team
The editorial team at ProProfs Quizzes consists of a select group of subject experts, trivia writers, and quiz masters who have authored over 10,000 quizzes taken by more than 100 million users. This team includes our in-house seasoned quiz moderators and subject matter experts. Our editorial experts, spread across the world, are rigorously trained using our comprehensive guidelines to ensure that you receive the highest quality quizzes.
Learn about Our Editorial Process
| By Chachelly
C
Chachelly
Community Contributor
Quizzes Created: 513 | Total Attempts: 597,090
Questions: 24 | Attempts: 223

SettingsSettingsSettings
Block 3 Pace 6 Nov 2011 - Quiz

Pace quiz #6 for block 3 nov 2011


Questions and Answers
  • 1. 

    Dr. Kirera A 65-year-old male patient with history of gastric cancer was admitted to the hospital for operation to remove the tumor. A central venous line was ordered for preoperative nutritional and fluid balance.  The venous line was placed via the left subclavian vein and blood returned freely through the catheter. The chest X‐ray film taken immediately after the catheterization revealed a substantial right hemidiaphragmatic elevation.  The condition resolved within 2 days after the catheter was readjusted by withdrawing it for about 2cm. Which of the following nerve was likely compressed by the tip of the catheter to elicit this condition?  

    • A.

      Right vagus

    • B.

      Right recurrent laryngeal

    • C.

      Right phrenic

    • D.

      Left phrenic

    • E.

      Left recurrent laryngeal

    • F.

      Left vagus

    Correct Answer
    C. Right phrenic
    Explanation
    The condition of right hemidiaphragmatic elevation suggests that the catheter tip compressed the right phrenic nerve. The phrenic nerve innervates the diaphragm, which is responsible for breathing. Compression of the right phrenic nerve would result in paralysis or weakness of the right hemidiaphragm, leading to its elevation. Adjusting the catheter by withdrawing it resolved the condition, indicating that the compression on the right phrenic nerve was relieved.

    Rate this question:

  • 2. 

    A 40-year-old man was diagnosed with an enlarged left atrium without incidence of mitral valve insufficiency. Which of the following symptoms is the most likely to be associated his condition?    

    • A.

      Dysphagia

    • B.

      Dyspnea

    • C.

      Chest pain

    • D.

      Facial edema

    • E.

      Voice hoarseness

    Correct Answer
    A. Dysphagia
    Explanation
    Dysphagia refers to difficulty swallowing, which can occur when the left atrium is enlarged and compresses the esophagus. This compression can lead to the sensation of food getting stuck or difficulty in swallowing. It is not directly related to mitral valve insufficiency, which is why the patient in this case does not have that condition. The other symptoms listed are not typically associated with an enlarged left atrium.

    Rate this question:

  • 3. 

    Shown is a drawing of a section of lung.  The arrow is pointing to the nucleus of which cell type?

    • A.

      Dust cell

    • B.

      Type II pneumocyte

    • C.

      Smooth muscle cell

    • D.

      Endothelial cell

    • E.

      Type I pneumocyte

    Correct Answer
    D. Endothelial cell
    Explanation
    The correct answer is Endothelial cell. The drawing is of a section of lung, and the arrow is pointing to the nucleus of a specific cell type. Endothelial cells are the cells that line the blood vessels and capillaries in the lungs. They play a crucial role in gas exchange and the regulation of blood flow in the lungs.

    Rate this question:

  • 4. 

    Dr. Smolanoff You have been hired by a biotechnology company as a Physician-Consultant to look into a new drug that has been invented. You are told that this new drug will revolutionize the diet industry because it selectively activates uncoupling proteins that perform the same function as thermogenin. Which of the following statements accurately describe what you would expect  to see in the mitochondria of cells treated with this drug?  

    • A.

      The cells would be capable of making ATP at the same rate as untreated cells.

    • B.

      The membrane potential in the mitochondria would be normal

    • C.

      Co-treatment with oligomycin would restore normal mitochondrial function

    • D.

      Mitochondrial NADH and FADH2 consumption would be inhibited

    • E.

      Mitochondrial oxygen consumption would be much higher than normal

    Correct Answer
    E. Mitochondrial oxygen consumption would be much higher than normal
    Explanation
    When the drug selectively activates uncoupling proteins that perform the same function as thermogenin, it would lead to an increase in mitochondrial oxygen consumption. This is because uncoupling proteins allow protons to flow back into the mitochondrial matrix without going through ATP synthase, which dissipates the proton gradient and increases oxygen consumption. As a result, the cells treated with this drug would have a higher rate of oxygen consumption compared to untreated cells.

    Rate this question:

  • 5. 

    A post-operative patient on intravenous fluids develops lesions in the mouth (angular stomatitis). Urinalysis indicates a clinical deficiency of riboflavin. Which of the following TCA enzymes is most likely to be affected?  

    • A.

      Citrate synthase

    • B.

      Fumarase

    • C.

      Malate dehydrogenase

    • D.

      Succinate dehydrogenase

    • E.

      Isocitrate dehydrogenase

    Correct Answer
    D. Succinate dehydrogenase
    Explanation
    Succinate dehydrogenase is the correct answer because riboflavin is a cofactor for this enzyme. Riboflavin, also known as vitamin B2, is necessary for the function of succinate dehydrogenase, which is an enzyme involved in the citric acid cycle (TCA cycle). Deficiency of riboflavin can lead to impaired activity of succinate dehydrogenase, resulting in various symptoms including angular stomatitis.

    Rate this question:

  • 6. 

    You are given a test tube by your biochemistry professor and asked to identify the purified enzyme it contains. You run a series of biochemical tests and determine that the enzyme requires acetyl CoA for activity, is inhibited by ATP, NADH and succinyl CoA and positively regulated by oxaloacetate. When you hand in the report, what enzyme did you conclude your professor gave you?  

    • A.

      Isocitrate dehydrogenase

    • B.

      Fumarase

    • C.

      Citrate synthase

    • D.

      Pyruvate dehydrogenase

    • E.

      Malate dehydrogenase

    Correct Answer
    C. Citrate synthase
    Explanation
    Based on the information provided, the enzyme in the test tube can be concluded to be Citrate synthase. This is because Citrate synthase requires acetyl CoA for activity, is inhibited by ATP, NADH, and succinyl CoA, and is positively regulated by oxaloacetate. None of the other enzymes listed have all these characteristics.

    Rate this question:

  • 7. 

    Dr. Bellot A 35 year old woman with a history of dyspnea and chest pain is shown by x-ray to have right ventricular enlargement and is clinically going into right heart failure. Primary pulmonary hypertension is suspected.  An intrapulmonary pathological finding characteristic of this condition is:    

    • A.

      Bullae formation

    • B.

      Plexiform arteriopathy

    • C.

      Alveolar fibrosis

    • D.

      Absence of pulmonary arterioles

    • E.

      Destruction of alveolar septa

    Correct Answer
    B. Plexiform arteriopathy
    Explanation
    Primary pulmonary hypertension is a condition characterized by increased blood pressure in the pulmonary arteries. Plexiform arteriopathy is a pathological finding commonly seen in primary pulmonary hypertension. It refers to the formation of complex vascular structures within the pulmonary arterioles, which can lead to obstruction of blood flow and further contribute to right ventricular enlargement and right heart failure.

    Rate this question:

  • 8. 

    Your patient was a 72 -year-old classic “pink puffer” with a chronic, dry cough and marked weight loss. He was obviously having difficulty breathing. The pathophysiological alteration in his lungs which lead to obstructive features clinically is    

    • A.

      Loss of recoil due to alveolar wall destruction

    • B.

      Chronic bronchial inflammation

    • C.

      Irreversible damage to bronchial walls

    • D.

      Replacement of alveolar wall by fibrous tissue

    • E.

      Narrowing of the bronchial lumen

    Correct Answer
    A. Loss of recoil due to alveolar wall destruction
    Explanation
    The correct answer is "loss of recoil due to alveolar wall destruction." In this patient, the chronic, dry cough and weight loss are indicative of chronic obstructive pulmonary disease (COPD). COPD is characterized by the destruction of alveolar walls, leading to a loss of elastic recoil in the lungs. This loss of recoil results in air trapping and difficulty breathing, which are the obstructive features seen clinically. Chronic bronchial inflammation and irreversible damage to bronchial walls may contribute to COPD, but the primary pathophysiological alteration is the loss of recoil due to alveolar wall destruction. The other options, such as replacement of alveolar wall by fibrous tissue and narrowing of the bronchial lumen, are not consistent with the given clinical presentation.

    Rate this question:

  • 9. 

    Dr. Beevers A 56-year-old man recently diagnosed with heart failure is started on a therapeutic regimen that includes the drugs furosemide, digoxin, and captopril.  What is the mechanism of action of the loop diuretic member of this regimen?  

    • A.

      Inhibition of Na+/K+ ATPase

    • B.

      Blockade of L-type Ca2+ channels

    • C.

      Stimulation of guanylyl cyclase

    • D.

      Inhibition of NKCC2

    • E.

      Inhibition of NCC

    Correct Answer
    D. Inhibition of NKCC2
    Explanation
    Loop diuretics, such as furosemide, work by inhibiting the NKCC2 transporter in the thick ascending limb of the loop of Henle in the kidneys. This transporter is responsible for reabsorbing sodium, potassium, and chloride ions from the urine back into the bloodstream. By inhibiting NKCC2, loop diuretics increase the excretion of these ions in the urine, leading to increased urine volume and decreased fluid volume in the body. This mechanism of action is important in treating conditions like heart failure, where reducing fluid volume can help alleviate symptoms.

    Rate this question:

  • 10. 

    Dr. Shams A patient inhales as much air as he can (maximal inspiratory level) and then you connect him with a spirometer containing a gas mixture of air and helium. After a homogeneous distribution of helium between spirometer gas and patients’ lung volume, you are able to measure the patients’ lung volume from helium dilution. Which of the following lung volumes or capacities do you have to know in addition to above measured lung volume to be able to calculate residual volume of this patient?  

    • A.

      Tidal volume

    • B.

      Functional residual capacity

    • C.

      Vital capacity

    • D.

      Inspiratory capacity

    • E.

      Expiratory reserve volume

    Correct Answer
    C. Vital capacity
    Explanation
    To calculate the residual volume of the patient, you would need to know the vital capacity in addition to the measured lung volume from helium dilution. The residual volume is the amount of air that remains in the lungs after a maximal exhalation, and it is calculated by subtracting the vital capacity from the total lung capacity. Therefore, knowing the vital capacity is necessary to determine the residual volume.

    Rate this question:

  • 11. 

    A patient suffers from an acute asthma attack. The patient has no history of chronic obstructive pulmonary disease. What change in following respiratory parameters is associated with his asthma attack?  

    • A.

      FRC is increased

    • B.

      RV is increased

    • C.

      Vital capacity is decreased

    • D.

      FEV1 is decreased

    • E.

      TLC is decreased

    Correct Answer
    D. FEV1 is decreased
    Explanation
    During an acute asthma attack, the airways become inflamed and narrowed, leading to difficulty in breathing. This narrowing of the airways causes a decrease in the forced expiratory volume in one second (FEV1). FEV1 is a measure of the maximum amount of air that can be forcefully exhaled in one second, and it is commonly used to assess lung function. Therefore, a decrease in FEV1 is associated with an asthma attack. The other respiratory parameters listed are not directly affected by an asthma attack.

    Rate this question:

  • 12. 

    A gas mixture of several dry gases has a total pressure of 100 mm Hg and a total gas amount of 10 Mol. What is the partial pressure of one of the gases that is in the amount of 2 Mol?  

    • A.

      20 mm Hg

    • B.

      30 mm Hg

    • C.

      40 mm Hg

    • D.

      50 mm Hg

    • E.

      60 mm Hg

    Correct Answer
    A. 20 mm Hg
    Explanation
    The partial pressure of a gas in a mixture is directly proportional to the amount of that gas present. In this case, the total gas amount is 10 Mol and the amount of the gas in question is 2 Mol. Therefore, the partial pressure of this gas can be calculated as (2 Mol / 10 Mol) * 100 mm Hg = 20 mm Hg.

    Rate this question:

  • 13. 

    A patient is breathing air at sea level and has a respiratory exchange ratio of 0.7. The arterial blood values are: PO2 100 mm Hg PCO2 28 mm Hg pH 7.60   The alveolar-arterial PO2 difference of this patient is closest to which of following values in mmHg?    

    • A.

      25

    • B.

      20

    • C.

      15

    • D.

      5

    • E.

      10

    Correct Answer
    E. 10
    Explanation
    The alveolar-arterial PO2 difference is a measure of the difference between the partial pressure of oxygen in the alveoli (the air sacs in the lungs) and the arterial blood. A normal value for this difference is around 10-15 mmHg. In this case, the patient's arterial blood PO2 is 100 mmHg, which is within the normal range. Therefore, the alveolar-arterial PO2 difference is likely to be closer to the lower end of the normal range, which is 10 mmHg.

    Rate this question:

  • 14. 

    The relationship between volume and transmural pressure of a normal lung is illustrated by curve 2. Which curve or line of the figure represents this relationship for the lung of a patient with surfactant deficiency?

    • A.

      1

    • B.

      3

    • C.

      4

    Correct Answer
    C. 4
  • 15. 

    A patient is suffering from obstructive lung disease.  His chest wall and respiratory system compliances are measured as:
    • Chest wall compliance (CCW) = 200 ml / cm H2O
    • Lung compliance (CL) = 200 ml /cm H2O
    What is his respiratory system compliance (CRS) in ml / cm H2O?  

    • A.

      300

    • B.

      50

    • C.

      200

    • D.

      100

    • E.

      400

    Correct Answer
    D. 100
    Explanation
    The respiratory system compliance (CRS) is determined by the sum of the chest wall compliance (CCW) and the lung compliance (CL). In this case, the CCW is 200 ml/cm H2O and the CL is also 200 ml/cm H2O. Therefore, the CRS would be 200 ml/cm H2O + 200 ml/cm H2O = 400 ml/cm H2O.

    Rate this question:

  • 16. 

    A person on a high altitude expedition develops lung edema. Which of following values is most likely increased (in resting condition) as a result of his lung edema?  

    • A.

      Arterial PO2

    • B.

      Arterial PCO2

    • C.

      Mixed venous PO2

    • D.

      Mixed venous PCO2

    • E.

      Alveolar- arterial PO2 difference

    Correct Answer
    E. Alveolar- arterial PO2 difference
    Explanation
    Lung edema refers to the accumulation of fluid in the lungs, which can impair the exchange of gases. The alveolar-arterial PO2 difference represents the difference in oxygen levels between the alveoli (air sacs in the lungs) and the arterial blood. In a healthy individual, this difference is small because oxygen is efficiently transferred from the alveoli to the blood. However, in the presence of lung edema, the transfer of oxygen is impaired, leading to an increased alveolar-arterial PO2 difference. Therefore, the most likely increased value in this scenario would be the alveolar-arterial PO2 difference.

    Rate this question:

  • 17. 

    At which lung volume or lung capacity is the resistance of alveolar blood vessels (capillaries) is the highest:  

    • A.

      Residual volume

    • B.

      Functional residual capacity

    • C.

      60% of total lung capacity

    • D.

      Minimal lung volume

    • E.

      100% of total lung capacity

    Correct Answer
    E. 100% of total lung capacity
    Explanation
    At 100% of total lung capacity, the resistance of alveolar blood vessels is the highest. This is because at this lung volume or capacity, the alveoli are maximally expanded and the blood vessels are stretched, leading to increased resistance.

    Rate this question:

  • 18. 

    Barometric pressure at sea level is about 760 mm Hg. The world’s highest summit (Mt. Everest) is 29,035 ft. and the barometric pressure is 247 mm Hg. The inspired PO2 of a climber who was breathing 50% oxygen at the summit would be about:

    • A.

      65 mm Hg

    • B.

      100 mm Hg

    • C.

      40 mm Hg

    • D.

      150 mm Hg

    • E.

      200 mm Hg

    Correct Answer
    B. 100 mm Hg
    Explanation
    At higher altitudes, the barometric pressure decreases. The barometric pressure at the summit of Mt. Everest is given as 247 mm Hg. The inspired PO2 (partial pressure of oxygen) can be calculated by multiplying the barometric pressure by the fraction of inspired oxygen (FiO2). In this case, the climber is breathing 50% oxygen, so the FiO2 is 0.5. Multiplying 247 mm Hg by 0.5 gives us 123.5 mm Hg, which is the inspired PO2. However, since the question asks for an approximation, the closest option is 100 mm Hg.

    Rate this question:

  • 19. 

    Minute ventilation is controlled by numerous factors including blood gases, mechanical receptors, body temperature, and voluntary effort (input from cortex). Which of the following mechanisms produces the greatest minute ventilation?  

    • A.

      Decreased PO2 plus an increased PCO2

    • B.

      Hypotension

    • C.

      Hyperthermia

    • D.

      Voluntary effort

    • E.

      Exercise

    Correct Answer
    D. Voluntary effort
    Explanation
    Voluntary effort produces the greatest minute ventilation because it involves conscious control and active participation of the individual in increasing their breathing rate and depth. This can result in a significant increase in the amount of air exchanged in the lungs per minute, leading to a higher minute ventilation. The other factors mentioned, such as decreased PO2 plus increased PCO2, hypotension, hyperthermia, and exercise, may also affect minute ventilation to some extent, but voluntary effort has the greatest impact.

    Rate this question:

  • 20. 

    The airway resistance of a person depends on lung volume. At which lung volume or lung capacity is the airway resistance the lowest?  

    • A.

      Residual volume

    • B.

      100% of total lung capacity

    • C.

      Functional residual capacity

    • D.

      60% of total lung capacity

    • E.

      Minimal lung volume

    Correct Answer
    B. 100% of total lung capacity
    Explanation
    At 100% of total lung capacity, the airway resistance is the lowest. This is because at this lung volume, the airways are maximally expanded, allowing for smooth and easy airflow.

    Rate this question:

  • 21. 

    A 65-year-old male visits his family practitioner for a yearly examination. Measurement of his blood pressure          reveals a systolic pressure of 190 mm Hg and a diastolic pressure of 100 mm Hg. His heart rate is 74/min and          pulse pressure is 90 mm Hg. A decrease in which of the following is the most likely explanation for the high pulse          pressure?

    • A.

      Arterial compliance

    • B.

      Cardiac output

    • C.

      Myocardial contractility

    • D.

      Stroke volume

    • E.

      Total peripheral resistance

    Correct Answer
    A. Arterial compliance
    Explanation
    The correct answer is A. A decrease in arterial compliance indicates that the arterial wall is stiffer (i.e., less
    distensible). When the compliance of the arterial system decreases, the rise in arterial pressure becomes
    greater for a given stroke volume pumped into the arteries. In the normal young adult, the systolic blood
    pressure is about 120 mm Hg and the diastolic blood pressure is about 80 mm Hg. Because the pulse pressure
    is the difference between the systolic and diastolic blood pressures, the normal pulse pressure is about 40 mm
    Hg in a healthy young adult. However, in older adults the pulse pressure sometimes increases as much as two
    times normal because the arteries become hardened by arteriosclerosis.

    The cardiac output (choice B) itself has no direct effect on the pulse pressure; however, if a decrease in
    cardiac output is associated with a decrease in stroke volume, the pulse pressure would be expected to
    decrease.

    A decrease in myocardial contractility (choice C) would be expected to decrease stroke volume, and therefore
    cause the pulse pressure to decrease.

    A decrease in stroke volume (choice D) causes the pulse pressure to decrease because a smaller amount of
    blood enters the arterial system with each heartbeat, and the rise and fall of pressure during systole and - 1 -
    diastole is decreased.

    A decrease in total peripheral resistance (choice E), i.e., vasodilation, does not have a significant effect on the
    pulse pressure of the major arteries under normal conditions.

    Rate this question:

  • 22. 

    A 54-year-old male is seen in clinic with complaints of palpitations and light-headedness. Physical examination is remarkable for a heart rate of greater than 200 beats per minute and a blood pressure of 75/40 mm Hg. What         adjustments have probably occurred in the cardiac cycle?  

    • A.

      Diastolic time has decreased and systolic time has increased

    • B.

      Diastolic time has decreased but systolic time has decreased more

    • C.

      Systolic time has decreased and diastolic time has increased

    • D.

      Systolic time has decreased but diastolic time has decreased more

    • E.

      Systolic time has decreased but diastolic time has not changed

    Correct Answer
    D. Systolic time has decreased but diastolic time has decreased more
    Explanation
    The correct answer is D. Under normal conditions, one-third of the cardiac cycle is spent in systole and
    two-thirds spent in diastole. As heart rate increases dramatically, the time spent in diastole falls precipitously
    but the time spent in systole falls only slightly.

    A large increase in heart rate must produce a decrease in both diastole and systole (compare with choice A).

    The major change with increased heart rate is in diastole, not systole (compare with choice B).

    Heart rate cannot increase if diastolic time increases (choice C).

    An increase in heart rate must be accompanied by a decrease in diastolic time (compare with choice E).

    Rate this question:

  • 23. 

    In a tissue capillary, the interstitial hydrostatic pressure is 2 mm Hg, the capillary hydrostatic pressure is 25 mm         Hg and the interstitial oncotic pressure is 7 mm Hg. If the net driving force across the capillary wall is 3 mm Hg         favoring filtration, what is the capillary oncotic pressure?

    • A.

      21 mm Hg

    • B.

      23 mm Hg

    • C.

      24 mm Hg

    • D.

      25 mm Hg

    • E.

      27 mm Hg

    Correct Answer
    E. 27 mm Hg
    Explanation
    The correct answer is E. The net driving force for fluid across a capillary wall is calculated by the following:

    driving force = (hydrostaticc− (hydrostatici) − (oncoticc - oncotici)

    where:

    hydrostatici = interstitial hydrostatic pressure

    hydrostaticc = capillary hydrostatic pressure

    oncotici = interstitial oncotic pressure

    oncoticc = capillary oncotic pressure

    Substituting the values in the question stem: 3 = (25 - 2) - (x - 7). Simplifying, 3 = 23 - x + 7, therefore = 27.

    Rate this question:

  • 24. 

    A balloon-tipped catheter is placed into a small branch of the pulmonary artery in a patient. The lumen of the          catheter opens distal to the balloon. The pressure measured from the catheter with the balloon deflated is 25/8          mm Hg. When the balloon is inflated, the pressure is 7 mm Hg and non-pulsatile. Which of the following          pressures is being approximated when the balloon is inflated?

    • A.

      Left atrial pressure

    • B.

      Left ventricular end diastolic pressure

    • C.

      Left ventricular peak systolic pressure

    • D.

      Pulmonary artery pressure

    • E.

      Right atrial pressure

    Correct Answer
    A. Left atrial pressure
    Explanation
    The correct answer is A. When the balloon is deflated, the catheter simply measures the pulmonary artery
    pressure (choice D), which is pulsatile with systolic/diastolic values of 25/8 mm Hg. When the balloon is inflated,
    the catheter is "wedged" in a small branch of the pulmonary artery and the pressure that is measured is called
    the "pulmonary wedge pressure." Because inflation of the balloon obstructs all blood flow in the artery branch,
    the blood vessels distal to the point of obstruction also have no flow. One can think of these distal vessels as
    physical extensions of the catheter, as they allow blood pressure to be measured on the other side of the
    pulmonary circulation, i.e., in the left atrium. The pulmonary wedge pressure is usually a few mm Hg higher
    compared to the left atrial pressure, but the general opinion is that pulmonary wedge pressure is a reflection of
    events in the left atrium. It is usually not feasible to measure left atrial pressure directly in the normal human
    being because it is difficult to pass a catheter retrograde through the aorta and left ventricle. Therefore, the
    pulmonary wedge pressure provides an important clinical estimate of left atrial pressure. Be aware that
    pulmonary wedge pressure may also be called pulmonary capillary wedge pressure, pulmonary arterial wedge
    pressure, or simply wedge pressure.

    In many instances, the pulmonary wedge pressure can provide a reasonable estimate of left ventricular end
    diastolic pressure (choice B). However, a notable exception is during mitral stenosis, in which the pressure in
    the left atrium (and therefore, the pulmonary wedge pressure) is much higher than the left ventricular end
    diastolic pressure because of the high resistance to blood flow through the stenosed valve.

    The left ventricular peak systolic pressure (choice C) occurs when the mitral valve is closed, making it
    impossible to be approximated using a catheter in the pulmonary artery.

    The right atrial pressure (choice E) cannot be measured or approximated from a catheter in the pulmonary
    artery.

    Rate this question:

Quiz Review Timeline +

Our quizzes are rigorously reviewed, monitored and continuously updated by our expert board to maintain accuracy, relevance, and timeliness.

  • Current Version
  • Mar 17, 2023
    Quiz Edited by
    ProProfs Editorial Team
  • Dec 03, 2011
    Quiz Created by
    Chachelly
Back to Top Back to top
Advertisement
×

Wait!
Here's an interesting quiz for you.

We have other quizzes matching your interest.